LSAT Logic Games : Linear Games

Study concepts, example questions & explanations for LSAT Logic Games

varsity tutors app store varsity tutors android store

Example Questions

1 2 43 44 45 46 47 48 49 51 Next →

Example Question #61 : Solving Four And Five Variable Logic Games

On Sunday, Marvin plans his dinners for the following 6 days (Monday-Saturday). He plans 6 main courses and 3 salads. Of the main courses, 3 are pasta dishes—Angel Hair, Fettuccine, and Spaghetti—and 3 are meat dishes—Steak, Chicken Breast, and Pork Loin. The 3 salads are Caesar Salad, Greek Salad, and Garden Salad. All of the following statements are true about the meals Marvin has planned for the week:

  • Each main course and each salad will be served exactly once during the week
  • A pasta dish is never served on the same night as a meat dish
  • Salad is always paired with either a pasta dish or a meat dish

  • The 3 salads are not always paired with a meat dish nor are they always paired with a pasta dish

  • No two meat dishes are served on consecutive days

  • Both Wednesday and Friday have pasta dishes

  • Both Monday and Thursday have salads with dinner

Which of the following statements would not be possible given that the above information is all true?

Possible Answers:

Friday night has Spaghetti dinner with Greek Salad.

Neither Pork Loin nor Chicken Breast will be served on Monday.

Neither Monday nor Tuesday has a pasta dinner.

Neither Pork Loin nor Chicken Breast will be served on Tuesday.

Niether Tuesday nor Saturday has a salad.

Correct answer:

Neither Monday nor Tuesday has a pasta dinner.

Explanation:

If all the given information is true, either Monday or Tuesday night must have a pasta dinner.  Since you know that:

  • No two meat dishes are planned on consecutive days

  • Both Wednesday and Friday have pasta dinners

you can determine that meat dishes must be served on Thursday and Saturday, which means there is only one remaining night for a meat dish to be served.  If Monday AND Tuesday did not have pasta dishes, they would both have to have meat dishes, which would be impossible.  (Also, this would violate the "no meat dishes on consecutive days" rule.)

Example Question #62 : Solving Four And Five Variable Logic Games

On Sunday, Marvin plans his dinners for the following 6 days (Monday-Saturday). He plans 6 main courses and 3 salads. Of the main courses, 3 are pasta dishes—Angel Hair, Fettuccine, and Spaghetti—and 3 are meat dishes—Steak, Chicken Breast, and Pork Loin. The 3 salads are Caesar Salad, Greek Salad, and Garden Salad. All of the following statements are true about the meals Marvin has planned for the week:

  • Each main course and each salad will be served exactly once during the week
  • A pasta dish is never served on the same night as a meat dish
  • Salad is always paired with either a pasta dish or a meat dish

  • The 3 salads are not always paired with a meat dish nor are they always paired with a pasta dish

  • No two meat dishes are served on consecutive days

  • Both Wednesday and Friday have pasta dishes

  • Both Monday and Thursday have salads with dinner

Which of the following pieces of information would allow you to determine whether the Chicken Breast would be eaten with a salad?

Possible Answers:

Spaghetti is served on Friday, Fettuccine is served with Greek salad.

A pasta dish is served on Monday.

Steak is served on Tuesday and Pork Loin is served later in the week than every pasta dish

The Caesar Salad is served with either Angel Hair Pasta or Pork Loin.

Angel Hair Pasta is served earlier in the week than Fettuccine or Spaghetti.

Correct answer:

Steak is served on Tuesday and Pork Loin is served later in the week than every pasta dish

Explanation:

You know that

  • No two meat dishes are planned on consecutive days

  • Both Wednesday and Friday have pasta dinners

  • Both Monday and Thursday have salads with dinner

By process of elimination, you can infer that Thursday and Saturday have meat dishes for dinner.  The answer choice "Steak is served on Tuesday, Pork Loin is served later in the week than every pasta dish" tells you that Pork Loin is served on Saturday, which means the only meat dish left to be eaten on Thursday would be Chicken Breast, and it would be eaten with a salad.  The other answer choices do not provide enough relevant information to help you determine this.

Example Question #61 : Four & Five Variable

A popular band is preparing for a nationwide music tour.  The tour must include the following five states: California, Florida, Illinois, New York, and Texas.  The tour will conclude in one state before moving to the next state.  The following restrictions apply to the tour dates:

The tour must start in New York or California.

Texas cannot be the final state.

Texas and Illinois must be consecutive, with either coming first.

Florida must be after New York.

New York and California must not be consecutive to each other.

How many possible positions in the tour sequence can New York be in?

Possible Answers:

Two

Five

Three

One

Four

Correct answer:

Two

Explanation:

New York cannot be second.  If New York is second, California must be first and the two cannot be consecutive.  New York cannot be third because then California would be first, and either Texas and Illinois would not be consecutive, or Florida would not be after New York.  New York also cannot be fifth because Florida must come after New York. Thus, New York can be first or fourth only.

Example Question #61 : Four & Five Variable

Over the course of four weeks, each of seven television programs will be aired to a focus group for help in coming up with a title. The programs are identified as G, H, J, K, L, M, and O. A different pair of these programs will be aired each week. Exactly one of the programs will be a member of two of these four pairs. The following constraints must be observed:

J is not aired during a given week unless H is aired during the week immediately preceding J’s airing.

The show that is aired during two of the weeks is aired during week 4 but is not aired during week 3.

G is not aired during a given week unless either J or else O is also aired during that week.

K is aired during one of the first two weeks.

O is one of the programs aired during week 3.

Which one of the following is a pair of programs that CANNOT be aired during the same week as each other?

Possible Answers:

L and M

H and M

J and O

K and L

H and K

Correct answer:

J and O

Explanation:

Best to start with the rule that G must be aired with either J or O. From this we can infer that J and O can never be aired together. If J and O are aired together, the pairing of J and O must occur in week 3, but then neither J nor O could be doubled, and one of J and O would have to be doubled in order to go with G. That line of reasoning indicates that the credited response is “J and O.”

Example Question #65 : Solving Four And Five Variable Logic Games

Over the course of four weeks, each of seven television programs will be aired to a focus group for help in coming up with a title. The programs are identified as G, H, J, K, L, M, and O. A different pair of these programs will be aired each week. Exactly one of the programs will be a member of two of these four pairs. The following constraints must be observed:

J is not aired during a given week unless H is aired during the week immediately preceding J’s airing.

The show that is aired during two of the weeks is aired during week 4 but is not aired during week 3.

G is not aired during a given week unless either J or else O is also aired during that week.

K is aired during one of the first two weeks.

O is one of the programs aired during week 3.

Which one of the following must be aired during week 2?

Possible Answers:

M

G

K

L

J

Correct answer:

J

Explanation:

In a question that asks what MUST be in a particular slot, it is extremely helpful to look at the answer tied to the acceptability question. In that question, we see that week 2 involves G and J. Since we know that week 2 MUST have a particular entity in it, we now know that this particular entity must be either G or J. Eliminate all other answer choices and simply do a trial-and-error test to determine whether G or J can be omitted from week 2. That test reveals that J must always be in week 2.

Example Question #66 : Solving Four And Five Variable Logic Games

Over the course of four weeks, each of seven television programs will be aired to a focus group for help in coming up with a title. The programs are identified as G, H, J, K, L, M, and O. A different pair of these programs will be aired each week. Exactly one of the programs will be a member of two of these four pairs. The following constraints must be observed:

J is not aired during a given week unless H is aired during the week immediately preceding J’s airing.

The show that is aired during two of the weeks is aired during week 4 but is not aired during week 3.

G is not aired during a given week unless either J or else O is also aired during that week.

K is aired during one of the first two weeks.

O is one of the programs aired during week 3.

Which one of the following CANNOT be the show that is aired during two of the weeks?

Possible Answers:

H

K

M

L

G

Correct answer:

G

Explanation:

If a show is aired during two weeks, then, according to the second rule, that show must be aired during week 4.  Making appropriate deductions (see below), we can see that G, J, and O cannot be aired in week 4.

Note:  H cannot be aired in weeks 2 or 3 due to the second rule.  J cannot be doubled since that would also require H to be doubled (no two doubles).  Therefore, J can be eliminated from weeks 1, 3, and 4.  Since G must be aired with either J or O, and since J and O cannot be doubled, G must be aired during week 2 or 3 but not week 1 or 4.

Example Question #501 : Linear Games

Over the course of four weeks, each of seven television programs will be aired to a focus group for help in coming up with a title. The programs are identified as G, H, J, K, L, M, and O. A different pair of these programs will be aired each week. Exactly one of the programs will be a member of two of these four pairs. The following constraints must be observed:

J is not aired during a given week unless H is aired during the week immediately preceding J’s airing.

The show that is aired during two of the weeks is aired during week 4 but is not aired during week 3.

G is not aired during a given week unless either J or else O is also aired during that week.

K is aired during one of the first two weeks.

O is one of the programs aired during week 3.

Which one of the following is a program that could be aired in any of the four weeks?

Possible Answers:

O

H

K

J

L

Correct answer:

L

Explanation:

This question can be effectively attacked by using deductions that are required to answer the other questions associated with this game.  Based on the rules and the deductions, the following entities have restrictions attached to them:  G, H, J, K, and O.  Accordingly, L is the correct answer.

Example Question #72 : Solving Four And Five Variable Logic Games

Over the course of four weeks, each of seven television programs will be aired to a focus group for help in coming up with a title. The programs are identified as G, H, J, K, L, M, and O. A different pair of these programs will be aired each week. Exactly one of the programs will be a member of two of these four pairs. The following constraints must be observed:

J is not aired during a given week unless H is aired during the week immediately preceding J’s airing.

The show that is aired during two of the weeks is aired during week 4 but is not aired during week 3.

G is not aired during a given week unless either J or else O is also aired during that week.

K is aired during one of the first two weeks.

O is one of the programs aired during week 3.

Which one of the following is a pair of shows that could be aired during the same week as each other?

Possible Answers:

M and O

H and J

G and H

K and O

H and O

Correct answer:

M and O

Explanation:

"G and H" can be eliminated because G must be paired with either J or O.  "J and H" can be eliminated because if J and H were paired, then H would appear in both weeks 1 and 2 and that violates the rules.  Neither H nor K can be aired in week 3.  That eliminates two answer choices:  "H and O" and "K and O."

M and O remains as the correct answer.

Example Question #72 : Four & Five Variable

Five basketball players - Adams, Griffin, Jones, Smith, and Washington - were ranked in five statistical categories: points, rebounds, assists, steals, and blocks.  Each player was the leader in one category and there were no ties.  No other players were considered for the purpose of these rankings.  The rankings of the players met the following conditions:

Smith ranked higher in points than in rebounds.

Griffin had a better ranking in rebounds than in assists and steals.

Adams was not the highest point scorer.

Washington was ranked last in blocks.

Which player could be the leader in potentially any category?

Possible Answers:

Smith

Adams

Washington

Griffin

Jones

Correct answer:

Jones

Explanation:

The correct answer is: Jones.

Each player is the leader of one category.  Smith ranked higher in points than in rebounds.  Therefore, Smith cannot be the leader in rebounds.  Griffin ranked higher in rebounds than in assists and steals.  Therefore, Griffin cannot be the leader in assists or steals.  Adams was not the highest point scorer.  Therefore, Adams cannot be the leader in points.  Washington ranked last in blocks.  Therefore, Washington cannot be the leader in blocks.  Thus, the correct answer is Jones.

1 2 43 44 45 46 47 48 49 51 Next →

Tired of practice problems?

Try live online LSAT prep today.

1-on-1 Tutoring
Live Online Class
1-on-1 + Class
Learning Tools by Varsity Tutors